LSAT and Law School Admissions Forum

Get expert LSAT preparation and law school admissions advice from PowerScore Test Preparation.

General questions relating to LSAT Logical Reasoning.
 eober
  • Posts: 107
  • Joined: Jul 24, 2014
|
#15567
Hi!

I am trying to get my head around how to approach Flaw in the Reasoning questions and I was wondering if in these types of questions we are not taking the information in the stimulus as being the only truth there is, as we do in must be true questions, would that be correct?

And also in one of the strengthen questions in the course book (page 4-55, question 13) the question is asking which one of the answer choices would be LEAST supported. Answer choice B states that: all of the centipedes that had previously been discovered were land dwellers. This Answer choice is accepted as strengthening the argument. However, isn't this answer choice similar to the flaw in the reasoning, in which what has occurred previously is assumed to provide support for what is occurring now? In other words, would this provide adequate support for the conclusion?

Any clarification and guidance on how to approach flaw in the reasoning questions is appreciated! :)
Thanks!
 Lucas Moreau
PowerScore Staff
  • PowerScore Staff
  • Posts: 216
  • Joined: Dec 13, 2012
|
#15569
Hello, eober,

Flaw in the Reasoning questions are part of the Prove Family! As you can see on page 1-6 of your first Full-Length Course book, Flaw questions should always be treated as if everything said in the stimulus is true. The flaw you are looking for in such questions is in the logic of the argument, the reasoning, not in the facts that the argument states.

As for the Strengthen question, that's the difference between Strengthen and Justify the Conclusion. With Strengthen questions, the correct answer choice doesn't need to prove the argument outright, it merely needs to make the argument more likely to be true. B would lend credence to the theory that these centipedes are land-dwellers, and so it would strengthen the conclusion.

For Strengthen questions, you don't need to worry about "adequate" support - just support! :-D

Hope that helps,
Lucas Moreau
 eober
  • Posts: 107
  • Joined: Jul 24, 2014
|
#15570
Helps a lot! Thank you so much Lucas!!

Get the most out of your LSAT Prep Plus subscription.

Analyze and track your performance with our Testing and Analytics Package.